Difference between revisions of "2008 AIME II Problems/Problem 9"
m (→Solution 2: typo fix) |
I like pie (talk | contribs) m (→Solution 2: Style, formatting) |
||
Line 11: | Line 11: | ||
Basically, the thing comes out to | Basically, the thing comes out to | ||
− | < | + | <center><math>a_{150} = (((5a + 10)a + 10)a + 10 \ldots) = 5a^{150} + 10 a^{149} + 10a^{149}+ \ldots + 10</math></center> |
− | a_{150} = (((5a + 10)a + 10)a + 10 | ||
− | </ | ||
Notice that | Notice that | ||
− | < | + | <center><math>10(a^{150} + \ldots + 1) = 10(1 + a + \ldots + a^6) = - 10(a^7) = - 10( - \sqrt {2}/2 - i\sqrt {2}/2)</math></center> |
− | 10(a^{150} + \ldots + 1) = 10(1 + a + \ldots + a^6) = - 10(a^7) = - 10( - \sqrt {2}/2 - i\sqrt {2}/2) | ||
− | </ | ||
Furthermore, <math>5a^{150} = - 5i</math>. So our final answer is | Furthermore, <math>5a^{150} = - 5i</math>. So our final answer is | ||
− | < | + | <center><math>5\sqrt {2} + 5(\sqrt {2} + 1) \approx 19.1 \Longrightarrow \boxed{019}</math></center> |
− | 5\sqrt {2} + 5(\sqrt {2} + 1) \approx 19.1 \Longrightarrow \boxed{019} | ||
== See also == | == See also == |
Revision as of 13:40, 19 April 2008
Problem
A particle is located on the coordinate plane at . Define a move for the particle as a counterclockwise rotation of radians about the origin followed by a translation of units in the positive -direction. Given that the particle's position after moves is , find the greatest integer less than or equal to .
Solution
Solution 1
Show periodic with steps, then invert twice. Template:Incomplete
Solution 2
Let the particle's position be represented by a complex number. The transformation takes to where and . We let and so that we want to find .
Basically, the thing comes out to
Notice that
Furthermore, . So our final answer is
See also
2008 AIME II (Problems • Answer Key • Resources) | ||
Preceded by Problem 8 |
Followed by Problem 10 | |
1 • 2 • 3 • 4 • 5 • 6 • 7 • 8 • 9 • 10 • 11 • 12 • 13 • 14 • 15 | ||
All AIME Problems and Solutions |